Proving Limit of Piecewise Function at x=0

  • MHB
  • Thread starter Harambe1
  • Start date
  • Tags
    Limits
In summary, for your function $f(x)$, if $\{x_n\}$ is a sequence of rational numbers tending to $0$, then $f(x_n) = 1$ for all $n$. If you can find a sequence of irrational numbers $\{x_n\}$ such that $\lim_{x\to0}f(x_n)$ is some number different from $1$, then $\lim_{x\to0}f(x)$ does not exist.
  • #1
Harambe1
5
0
Hi,

I'm struggling to prove that a limit ceases to exist as x tends to 0 for the following function:

\(\displaystyle f(x)=\begin{cases}\sin(\frac{1}{x}), & \text{if $x \notin \mathbb{Q}$} \\[3pt] 1, & \text{if $x \in \mathbb{Q}$} \\ \end{cases}\)

I've attempted a proof by contradiction, assuming the limit is $L$ but I'm not sure how to then use the epsilon-delta definition to prove that a limit doesn't exist.

Any tips appreciated. Thanks.
 
Physics news on Phys.org
  • #2
Welcome, Harambe! (Wave)

Let's consider what it means for $\lim_{x\to 0} f(x) = L$. By definition, it means that to every $\epsilon > 0$ there corresponds $\delta > 0$ such that for all $x\in \Bbb R$, $0 < \lvert x\rvert < \delta$ implies $\lvert f(x) - L\rvert < \epsilon$. Let's negate this: $\lim_{x\to 0} f(x) \neq L$ if there exists $\epsilon > 0$ such that for every $\delta > 0$, there is an $x\in \Bbb R$ such that $0 < \lvert x \rvert < \delta$ and $\lvert f(x) - L\rvert \ge \epsilon$. Now we have an $\epsilon-\delta$ formulation for the statement that $f(x)$ has no limit as $x \to 0$:

To every $L \in \Bbb R$, there corresponds an $\epsilon > 0$ such that for every $\delta > 0$, there exists an $x\in \Bbb R$ such that $0 < \lvert x\rvert < \delta$ and $\lvert f(x) - L\rvert \ge \epsilon$.

To prove the above statement, let $L\in \Bbb R$ and consider separate cases (a) $\lvert L \rvert \neq 1$ and (b) $\lvert L\rvert = 1$.
 
  • #3
Another way to tackle this problem would be to use sequences. Suppose that $\lim_{x\to0}f(x) = L.$ It follows that if $\{x_n\}$ is a sequence with $\lim_{n\to\infty}x_n = 0$ then $\lim_{n\to\infty}f(x_n) = L.$

For your function $f(x)$, if $\{x_n\}$ is a sequence of rational numbers tending to $0$, then $f(x_n) = 1$ for all $n$. It follows that if $\lim_{x\to0}f(x)$ exists then that limit must be $1$. If you can then find a sequence of irrational numbers $\{x_n\}$ such that $\lim_{x\to0}f(x_n)$ is some number different from $1$, it would follow that $\lim_{x\to0}f(x)$ does not exist.
 

What is a piecewise function?

A piecewise function is a mathematical function that is defined by different expressions or rules for different parts of its domain. This means that the function may have different formulas, or pieces, for different intervals of its input values.

What does it mean to prove the limit of a piecewise function at x=0?

Proving the limit of a piecewise function at x=0 means determining the value that the function approaches as the input value, in this case x, gets closer and closer to 0. This is also known as the limit of the function as x approaches 0.

Can a piecewise function have a limit at x=0?

Yes, a piecewise function can have a limit at x=0 as long as the left and right limits at x=0 are equal. This means that the function has a defined value at x=0 and is continuous at that point.

How do you prove the limit of a piecewise function at x=0?

To prove the limit of a piecewise function at x=0, you need to evaluate the left and right limits separately and then check if they are equal. If they are equal, then the limit at x=0 exists and is equal to the value of the left and right limits. If they are not equal, then the limit does not exist.

What are some common techniques used to prove the limit of a piecewise function at x=0?

Some common techniques used to prove the limit of a piecewise function at x=0 include direct substitution, algebraic manipulation, and the squeeze theorem. It is also important to carefully consider the domain and continuity of the function at x=0.

Similar threads

Replies
4
Views
1K
  • Calculus
Replies
4
Views
1K
  • Calculus
Replies
3
Views
747
Replies
2
Views
1K
Replies
9
Views
926
Replies
1
Views
1K
Replies
1
Views
1K
  • Calculus
Replies
5
Views
1K
Back
Top